2
0
Fork 0
mirror of https://github.com/MartinThoma/LaTeX-examples.git synced 2025-04-19 11:38:05 +02:00

added image

This commit is contained in:
Martin Thoma 2013-12-13 13:47:16 +01:00
parent 0ec4968e53
commit 74eb9d4712
8 changed files with 44 additions and 8 deletions

View file

@ -6,3 +6,4 @@ Datum | Uhrzeit
03.12.2013 | 11:00 - 12:00, 13:10 - 15:00
05.12.2013 | 15:50 - 17:00
12.12.2013 | 12:00 - 13:40, 16:23 - 18:22
13.12.2013 | 13:10 - 13:47

Binary file not shown.

View file

@ -34,7 +34,7 @@
\end{enumerate}
\end{aufgabe}
\begin{aufgabe}[Kompaktheit]\label{ub2:aufg3.1}
\begin{aufgabe}[Kompaktheit]\label{ub3:aufg1}
\begin{enumerate}[label=(\alph*)]
\item Ist $\text{GL}_n(\mdr) = \Set{A \in \mdr^{n \times n} | \det(A) \neq 0}$ kompakt?
\item Ist $\text{SL}_n(\mdr) = \Set{A \in \mdr^{n \times n} | \det(A) = 1}$ kompakt?

View file

@ -2,10 +2,6 @@
\section*{Übungsaufgaben}
\addcontentsline{toc}{section}{Übungsaufgaben}
\begin{aufgabe}\label{ub3:aufg1}
\todo{Todo}
\end{aufgabe}
\begin{aufgabe}[Zusammenhang]\label{ub4:aufg1}
\begin{enumerate}[label=(\alph*)]
\item Beweisen Sie, dass eine topologische Mannigfaltigkeit

View file

@ -2,6 +2,12 @@
\section*{Übungsaufgaben}
\addcontentsline{toc}{section}{Übungsaufgaben}
\begin{aufgabe}\label{ub5:aufg1}
\todo{Todo}
\begin{aufgabe}\label{ub7:aufg1}
Berechnen Sie die Homologiegruppen von $S^1$ und $S^2$, indem Sie
zu $S^1$ bzw. $S^2$ homöomorphe Simplizialkomplexe betrachten.
\end{aufgabe}
\begin{aufgabe}\label{ub7:aufg3}
Es sei $G$ eine topologische Gruppe und $e$ ihr neutrales
Element. Man beweise, dass $\pi_1(G,e)$ abelsch ist.
\end{aufgabe}

View file

@ -602,13 +602,20 @@ Haben wir Häufungspunkt definiert?}
\label{fig:satz-seifert-van-kampen}
\end{figure}
\begin{korollar}
\begin{korollar}\label{kor:12.5}%Bemerkung 12.5 aus Vorlesung
Sei $Z$ zusammenhängend und $f_0, \dots, f_1: Z \rightarrow Y$
Liftungen von $f$.
$\exists z_0 \in Z: f_0(z) = f_1(z) \Rightarrow f_0 = f_1$
\end{korollar}
\begin{figure}
\centering
\input{figures/commutative-diagram-2.tex}
\caption{Situation aus Korollar~\ref{kor:12.5}}
\label{fig:situation-kor-12.5}
\end{figure}
\begin{beweis}
Sei $T = \Set{z \in Z | f_0(z) = f_1(z)}$.

View file

@ -96,6 +96,10 @@
\end{enumerate}
\end{solution}
\begin{solution}[\ref{ub3:aufg1}]
\todo[inline]{Kommt noch.}
\end{solution}
\begin{solution}[\ref{ub4:aufg1}]
\begin{enumerate}[label=(\alph*)]
\item \textbf{Vor.:} Sei $M$ eine topologische Mannigfaltigkeit.\\
@ -154,3 +158,11 @@
\end{beweis}
\end{enumerate}
\end{solution}
\begin{solution}[\ref{ub7:aufg1}]
\todo[inline]{Kommt noch.}
\end{solution}
\begin{solution}[\ref{ub7:aufg3}]
\todo[inline]{Kommt noch.}
\end{solution}

View file

@ -0,0 +1,14 @@
\begin{tikzpicture}
\node (Z) at (0,0) {$Z$};
\node (Y) at (3,0) {$Y$};
\node (X) at (1.5,-1.5) {$X$};
\draw[->, above, dashed] (Z) to node {$\tilde{f}$} (Y);
\draw[->, below] (Z) to node {$f$} (X);
\draw[->, right] (Y) to node {$p$} (X);
\begin{scope}[xshift=1.3cm,yshift=-0.6cm]
\draw (0,0) -- (0.3,0.3);
\draw (0.1,0) -- (0.4,0.3);
\draw (0.2,0) -- (0.5,0.3);
\end{scope}
\end{tikzpicture}